Why not C?
Can someone explain by A is better than C? I was down to both answer choices but ruled out A.
Eugene on September 25 at 08:37PM
  • December 2003 LSAT
  • SEC4
  • Q13
1
Reply
Answer choice D
The study at the beginning of the passage concludes about reduction in the likelihood of stroke. ...
AndrewArabie on June 4 at 09:09PM
  • December 2003 LSAT
  • SEC4
  • Q20
1
Reply
Clarification on Answer and Question Type
Why is the correct answer A?
JLA on March 15, 2023
  • December 2003 LSAT
  • SEC4
  • Q22
1
Reply
Help
Hi chose answer choice C, but the correct answer is B. Can someone explain the stimulus? I don't ...
Abigail-Okereke on January 24, 2023
  • December 2003 LSAT
  • SEC4
  • Q15
3
Replies
Why is (C) incorrect/ issues with (E)
Hello - can you explain why (C) is incorrect? My issue with (E) is that the warmth-adapted beetle...
LSATMaxUser on November 11, 2022
  • December 2003 LSAT
  • SEC4
  • Q7
2
Replies
Clarification/Drawn Out
Would someone please help me diagram the premise/argument and the answer choices? I'm struggling ...
Julia96 on August 21, 2022
  • December 2003 LSAT
  • SEC4
  • Q22
1
Reply
Can someone explain the correct answer please?
I was on edge about the last sentence being the conclusion but I felt the first sentence was cont...
BennyB33 on June 26, 2022
  • December 2003 LSAT
  • SEC4
  • Q16
2
Replies
Answer A
Could you please explain why A is the correct answer?
yckim2180 on May 18, 2021
  • December 2003 LSAT
  • SEC4
  • Q20
1
Reply
Why is E correct?
Why is E correct?
Shiyi-Zhang on May 11, 2021
  • December 2003 LSAT
  • SEC4
  • Q7
2
Replies
Why C is wrong? and Why B is right?
need an explanation
Guile on April 15, 2021
  • December 2003 LSAT
  • SEC4
  • Q3
1
Reply
Why D is right?
need an explanation
Guile on April 14, 2021
  • December 2003 LSAT
  • SEC4
  • Q21
1
Reply
Why is C wrong?
Hi Why is C wrong? Thanks
jingjingxiao11111@gmail.com on April 26, 2020
  • December 2003 LSAT
  • SEC4
  • Q14
1
Reply
Why is A wrong?
Hi Why is A wrong? Thanks
jingjingxiao11111@gmail.com on April 15, 2020
  • December 2003 LSAT
  • SEC4
  • Q10
2
Replies
Why is Answer E Correct
Why is this the conclusion... and why is it not the sentence that comes after thus
zacharylouiskane@gmail.com on April 10, 2020
  • December 2003 LSAT
  • SEC4
  • Q16
1
Reply
Clarification
My take after reading the stimulus initially was that the teacher makes a claim then illustrates ...
gharibiannick on March 21, 2020
  • December 2003 LSAT
  • SEC4
  • Q3
1
Reply
Conclusion
Initially, I thought "it is clear that humans are still biologically adapted to a diet of wild fo...
gharibiannick on March 21, 2020
  • December 2003 LSAT
  • SEC4
  • Q18
1
Reply
Necessary assumptoin
I son understand why A is not the correct answer. There is a change in the stimulus from right to...
Kath on September 30, 2019
  • December 2003 LSAT
  • SEC4
  • Q19
4
Replies
Answer to question 8
Why is the answer "E" and not "D"? Thanks for the help!
Dannyd7 on September 12, 2015
  • December 2003 LSAT
  • SEC4
  • Q8
1
Reply
Cacti and oranges
Ok I get ur explanation ... But I need help understanding this: 1country -most- ~DC (~H) or ~Do...
Lily on August 31, 2013
  • December 2003 LSAT
  • SEC4
  • Q25
1
Reply